Difference between revisions of "2014 AMC 12A Problems/Problem 22"

(Created page with "{{duplicate|2014 AMC 12A #22 and 2014 AMC 10A #25}} ==Problem== The number <math>5^{867}</math> is between <math>2^{2013}</ma...")
 
(Hmm, well, just redirecting to 2014 AMC 10A Problems/Problem 25 would be better.)
 
Line 1: Line 1:
{{duplicate|[[2014 AMC 12A Problems|2014 AMC 12A #22]] and [[2014 AMC 10A Problems|2014 AMC 10A #25]]}}
+
#REDIRECT[[2014 AMC 10A Problems/Problem 25]]
==Problem==
 
 
 
The number <math>5^{867}</math> is between <math>2^{2013}</math> and <math>2^{2014}</math>.  How many pairs of integers <math>(m,n)</math> are there such that <math>1\leq m\leq 2012</math> and <cmath>5^n<2^m<2^{m+2}<5^{n+1}?</cmath>
 
<math>\textbf{(A) }278\qquad
 
\textbf{(B) }279\qquad
 
\textbf{(C) }280\qquad
 
\textbf{(D) }281\qquad
 
\textbf{(E) }282\qquad</math>
 
 
 
==Solution==
 
 
 
Between any two powers of 5 there are either 2 or 3 powers of 2 (because <math>2^2<5^1<2^3</math>). Consider the intervals <math>(5^0,5^1),(5^1,5^2),\dots (5^{866},5^{867})</math>. We want the number of intervals with 3 powers of 2.
 
 
 
From the given that <math>2^{2013}<5^{867}<2^{2014}</math>, we know that these 867 intervals together have 2013 powers of 2. Let <math>x</math> of them have 2 powers of 2 and <math>y</math> of them have 3 powers of 2. Thus we have the system
 
<cmath>x+y&=867</cmath><cmath>2x+3y&=2013</cmath>
 
from which we get <math>y=279</math>, so the answer is <math>\boxed{\textbf{(B)}}</math>.
 
 
 
(Solution by superpi83)
 
 
 
==See Also==
 
 
 
{{AMC10 box|year=2014|ab=A|num-b=24|after=Last Problem}}
 
{{AMC12 box|year=2014|ab=A|num-b=21|num-a=23}}
 
{{MAA Notice}}
 

Latest revision as of 16:41, 8 February 2014